LSAT and Law School Admissions Forum

Get expert LSAT preparation and law school admissions advice from PowerScore Test Preparation.

User avatar
 Dave Killoran
PowerScore Staff
  • PowerScore Staff
  • Posts: 5852
  • Joined: Mar 25, 2011
|
#44075
Complete Question Explanation
(The complete setup for this game can be found here: lsat/viewtopic.php?t=6217)

The correct answer choice is (B)

If only one store carries F that store must be X. That means that S carries J, O, and R. Also, V and Z must carry R and O, with R/O split between them (1 type each).

Answer choice (A) is incorrect because V carries only 1 type.

Answer choice (B) is the correct answer choice. S and T both carry R and O.

Answer choice (C) is incorrect because V carries only 1 type.

Answer choice (D) is incorrect because V carries only 1 type.

Answer choice (E) is incorrect because Z carries only 1 type.
User avatar
 AlwaysPreppin
  • Posts: 2
  • Joined: May 27, 2023
|
#101957
I may definitely be missing something here, but this question held me up way longer than it should have due to the following:

I see only one possibility for S and T given these Local conditions - which is J, O, and R in the S group and O and R in the T group. Therefore I identified answer choice B as a 'Must Be True' scenario, and due to the common trap of terms of truth, I sunk time into finding a 'Could Be True' answer (unsuccessfully).

Is answer choice B correct because Must Be True scenarios fall under the umbrella of Could Be True scenarios? Or am I missing an alternate possibility for S and/or T?
 Adam Tyson
PowerScore Staff
  • PowerScore Staff
  • Posts: 5153
  • Joined: Apr 14, 2011
|
#101960
Yes indeed, AlwaysPreppin, anything that must be true IS something that could be true! The only time this would be a wrong answer is when the question asks "which of the following could be true but is not necessarily true?"
User avatar
 AlwaysPreppin
  • Posts: 2
  • Joined: May 27, 2023
|
#101961
Adam Tyson wrote: Sun May 28, 2023 9:45 am Yes indeed, AlwaysPreppin, anything that must be true IS something that could be true! The only time this would be a wrong answer is when the question asks "which of the following could be true but is not necessarily true?"
Awesome, thanks!

Get the most out of your LSAT Prep Plus subscription.

Analyze and track your performance with our Testing and Analytics Package.